Your-Doctor
Multiple Choice Questions (MCQ)


Quiz Categories Click to expand

Category: Medicine--->Neurology
Page: 5

Question 21# Print Question

A 74-year-woman consults you because of tremor and difficulty completing her daily tasks on time. She has hypertension and takes hydrochlorothiazide 25 mg every morning. She does not smoke and uses alcohol infrequently. On examination, her BP is 126/84; her vital signs are otherwise unremarkable. Eye movements are normal as are her reflexes and motor strength. She moves slowly; her timed get-up-and-go test takes 24 seconds (normal 10 seconds). She has a slow resting tremor with a frequency of about 3 per second; the tremor is more prominent on the right than the left. The tremor decreases with intentional movement. Her handwriting has deteriorated and is small and crabbed.

Which therapy is most likely to improve her functional disabilities? 

A. Switching her antihypertensive to propranolol 20 mg po bid
B. Benztropine mesylate 0.5 mg po tid
C. Lorazepam 0.5 mg po tid
D. Ropinirole beginning at a dose of 0.25 mg tid
E. Carbidopa/levodopa beginning at a dose of one-half of a 25 mg/100 mg tablet tid


Question 22# Print Question

A 72-year-old woman is found unconscious at home by her daughter. The daughter last spoke to her mother 1 day previously, at which time her mother seemed fine. The patient has diabetes, hypertension, atrial fibrillation, and chronic back pain. Her medications include metformin, lisinopril, warfarin, and oxycodone. On examination her blood pressure is 167/70, pulse 48 beats/minute, respiratory rate 12 breaths/minute and irregular, and temperature 37.2°C (98.9°F). There are no signs of trauma. Neck issupple. The patient does not respond to verbal stimuli. Pupils are equally reactive to light. The oculocephalic reflex (doll’s eye maneuver) is normal. On applying firm pressure to the orbital rim, the patient flexes her right arm, but does not move her left arm.

Which of the following is the most likely cause of her condition?

A. Hypoglycemia
B. Narcotic overdose
C. Lacunar infarct in the right internal capsule
D. Acute subdural hematoma
E. Anterior cerebral artery embolism


Question 23# Print Question

A 37-year-old factory worker develops increasing weakness in the legs; coworkers have noted episodes of transient confusion. The patient has bilateral foot drop and atrophy; mild wrist weakness is also present. His CBC shows an anemia with hemoglobin of 9.6 g/dL; examination of the peripheral blood smear shows basophilic stippling.

Which of the following is the most likely cause of this patient’s symptoms? 

A. Amyotrophic lateral sclerosis
B. Lead poisoning
C. Overuse syndrome
D. Myasthenia gravis
E. Alcoholism


Question 24# Print Question

A 53-year-old woman presents with increasing weakness, most noticeable in the legs. She has noticed some cramping and weakness in the upper extremities as well. She has more difficulty removing the lids from jars than before. She has noticed some stiffness in the neck but denies back pain or injury. There is no bowel or bladder incontinence. She takes naproxen for osteoarthritis and is on alendronate for osteoporosis. She smokes one pack of cigarettes daily. The general physical examination reveals decreased range of motion in the cervical spine. On neurological examination, the patient has 4/5 strength in the hands with mild atrophy of the interosseous muscles. She also has 4/5 strength in the feet; the weakness is more prominent in the distal musculature. She has difficulty with both heel walking and toe walking. Reflexes are hyperactive in the lower extremities. Sustained clonus is demonstrated at the ankles.

What is the best next step in her management? 

A. Obtain MRI scan of the head
B. Begin riluzole
C. Obtain MRI scan of the cervical spine
D. Check muscle enzymes including creatine kinase and aldolase
E. Refer for physical therapy and gait training exercises


Question 25# Print Question

A 73-year-old man has had three episodes of visual loss in the right eye. The episodes last 20 to 30 minutes and resolve completely. He describes the sensation as like a window shade being pulled down in front of the eye. He has a history of hypertension and tobacco use. He denies dyspnea, chest pain, palpitations, or unilateral weakness or numbness. On examination the patient appears healthy; his vital signs are normal and the neurological examination is unremarkable. An ECG shows normal sinus rhythm without evidence of ischemia or hypertrophy. Initial laboratory studies are normal. Both noncontrast CT scan of the head and MR scan of the brain are normal.

What is the best next step in this patient’s management? 

A. Begin anticoagulation with low-molecular-weight heparin and warfarin
B. Obtain an echocardiogram
C. Check for antiphospholipid antibodies and homocysteine levels
D. Order a carotid duplex ultrasonogram and begin antiplatelet therapy
E. Begin lamotrigine for probable nonconvulsive seizure




Category: Medicine--->Neurology
Page: 5 of 5